PEDS FINAL

Ace your homework & exams now with Quizwiz!

A child is brought into the emergency department. After assessing a child's airway, breathing, and circulation (ABCs), which action will the nurse take next? Assess level of consciousness. Obtain a full set of vital signs. Remove the child's clothing. Provide pain management.

Assess level of consciousness. Rationale: Once the ABCs are completed, the nurse's next step is to assess the child's level of consciousness or disability. This would be followed by removing the child's clothing and diaper (exposure) to assess for underlying signs of illness or injury. Next, full vital signs are taken while facilitating the family presence, followed by giving comfort by managing pain and providing emotional support. The acronym ABCDEFGHI is a useful reminder of the order of assessment: airway, breathing, circulation, disability, exposure, full vital signs, family presence, give comfort, head-to-toe assessment, inspect, and isolate.

A child is diagnosed with varicella. The parent states the child is "just miserable" and wants to know how best to make the child feel more comfortable. Which instruction(s) would the nurse give this parent? Select all that apply. "Keep only light clothing on your child." "Keep your child's fingernails short so scratching will not disturb vesicles." "You can administer acetaminophen to help with fever and pain." "Administer prescribed topical steroid ointment to reduce inflammation." "Place the child in a soothing tepid bath."

not sure if these all all correct "Keep only light clothing on your child." "Keep your child's fingernails short so scratching will not disturb vesicles." "Administer prescribed topical steroid ointment to reduce inflammation." "Place the child in a soothing tepid bath." The use of aspirin in children with chickenpox has been associated with Reye's syndrome

In the emergency room, the nurse is assessing a toddler who is currently being treated for a radius fracture and has a history of multiple fractures. The assessment reveals short stature, blue sclera, and no bruising or swelling at the fracture site. The nurse suspects: child abuse (child maltreatment). attention deficit hyperactivity disorder. lack of parental supervision. osteogenesis imperfecta.

osteogenesis imperfecta. Rationale: Children with osteogenesis imperfecta often have blue sclera and fractures often lack swelling or bruising at the site. Child abuse (child maltreatment) or lack of parental supervision might be valid concerns if the signs and symptoms of osteogenesis imperfecta were absent. Attention deficit hyperactivity disorder alone would not cause multiple fractures.

Which piece of equipment is most helpful in determining airway obstruction in the client with asthma? A nebulizer An inhaler A peak flow meter An incentive spirometer

peak flow meter Rationale: The peak flow meter provides the most reliable early sign of an asthma episode. Most episodes begin gradually, and a drop in peak flow can alert the client to begin medications before symptoms actually are noticeable. A nebulizer and inhaler treat symptoms. An incentive spirometer is used for lung expansion, especially after surgery.

In developing a plan of care for the child diagnosed with rheumatic fever, the nursing intervention that takes highest priority for this child is to: monitor the C-reactive protein and erythrocyte sedimentation rate (ESR) levels. promote rest periods and bed rest. provide age-appropriate diversional activities. position the child to relieve joint pain.

promote rest periods and bed rest. Rationale: As long as the rheumatic process is active, progressive heart damage is possible. To prevent heart damage, bed rest is essential to reduce the heart's workload. Laboratory tests for ESR and C-reactive protein can be used to evaluate disease activity and guide treatment, but they do not improve the child's health. The child's comfort is important, so it is essential to relieve joint pain and prevent injury with padded bed rails. But these measures are less important than rest when it comes to preventing long-term complications such as residual heart disease.

Stage of development of toddlers as theorized by Erikson

Autonomy vs Shame & Doubt

The nurse is assessing a child diagnosed with Cushing disease. Which statement by the parents demonstrates a need for further teaching? "This disorder is most likely due to an infection my child had recently." "My child may experience excessive hair growth and rapid weight gain." "We need to pay close attention to any wounds our child gets to monitor for adequate healing." "My child's round, full face appearance is reversible with appropriate treatment."

"This disorder is most likely due to an infection my child had recently." Rationale: A round, full face (moon face), rapid weight gain, excessive hair growth and poor wound healing are all seen in Cushing disease. Cushingoid appearance is reversible with appropriate treatment. The most common cause of Cushing disease is long-term corticosteroid therapy or a pituitary adenoma, not an infection.

The nurse is reviewing discharge instructions with the parents of a 3-year-old child with a humeral fracture and a cast in place. Which statement by the parents requires further follow up by the nurse? "We have a hair dryer to use on the cool setting for itching." "We should avoid long periods outside in hot weather." "We will not immerse the cast in water." "We will provide some small toys to play with during recovery."

"We will provide some small toys to play with during recovery." Rationale: The nurse should follow up on the statement by the parents, "We will provide some small toys to play with during recovery," because the child may insert the small toys into the cast. Nothing should be inserted into the cast at any time. The statements, "We will not immerse the cast in water," "We have a hair dryer to use on the cool setting for itching," and "We should avoid long periods outside in hot weather," are all appropriate statements regarding cast care that do not require further follow up by the nurse.

The age at which a toddler usually runs

2 years old

When can infants sit independently?

9 months

The nurse is assessing the respiratory system of a newborn. Which anatomic differences place the infant at risk for respiratory compromise? Select all that apply. a. The nasal passages are narrower. b. The trachea and chest wall are less compliant. c. The bronchi and bronchioles are shorter and wider. d. The larynx is more funnel shaped. e. The tongue is smaller. f. There are significantly fewer alveoli.

A, D, F The nasal passages are narrower; the larynx is a more funnel shape; there are significantly lower alveoli

Which developmental milestone would the nurse expect an 11-month-old infant to have achieved? A. Sitting independently B. Walking independently C. Building a tower of four cubes D. Turning a doorknob

A. Sitting independently Rationale: Infants typically sit independently, without support, by age 8 months. Walking independently may be accomplished as late as age 15 months and still be within the normal range. Few infants walk independently by age 11 months. Building a tower of three or four blocks is a milestone of an 18-month-old. Turning a doorknob is a milestone of a 24-month-old.

The nurse is teaching an 8-year-old child the steps for using an epinephrine auto-injector. Place the steps for administering the medication with this device in the order the child will perform them. Use all options. A. Hold the auto-injector at a right angle to your thigh. B. Hold your leg steady. C. Place the orange tip of the auto-injector in the middle of your thigh. D. Push the auto-injector firmly until a "click" is heard.

B, A, C, D B. Hold your leg steady A. Hold the auto-injector at a right angle to your thigh. C. Place the orange tip of the auto-injector in the middle of your thigh. D. Push the auto-injector firmly until a "click" is heard.

The parents of a 10-year-old boy report they are having problems with their son. The child's mother reports her son is not a talented athlete but her husband continues to encourage him to play and try to excel. The child's father reports sports will help his son build character. What response by the nurse is most appropriate? A. "Encouraging involvement in sports can build valuable skills for a child." B. "Although your son is not a talented athlete, continue to encourage him to try." C. "Perhaps another pursuit would be better suited for your son." D. "It is important not to let him quit without trying."

C Rationale: Children should be encouraged to try new things but, when they are faced with things they are not good at, the pressures can ultimately be counterproductive if they feel they are disappointing their parents or others of importance to them.

The nurse is assessing a 6-month-old infant. The parent asks when the soft area in the infant's head will go away. What is the best response by the nurse? A. "The area is called the anterior fontanel (fontanelle) and typically closes anytime up to 18 months of age." B. "Soft spots on the infant's head are called fontanelles, both of them should have closed by now." C. "The area is called a fontanel (fontanelle). They remain open to allow for rapid brain growth in the first months of life." D. "The soft spots are called fontanelles, they may stay open until your infant is 2 or 3 years old."

C. "The area is called the anterior fontanel and typically closes anytime between 9 and 18 months of age." Rationale: The anterior fontanel (fontanelle) typically closes by the age of 9 to 18 months. Fontanels (fontanelles) are soft areas on the skull that remain open in infancy to allow for rapid brain growth in the first months of life. This answer is a true statement but does not answer the parent's question.

2 most important interventions in correct order for a child suspected of having UTI

Culture Antibiotic therapy

A 6-month-old infant weighs 14.7 pounds (6.7 kg) during a scheduled check-up. The birth weight was 8 pounds (3600 g). What is the priority nursing intervention? a. Talking about solid food consumption b. discouraging daily fruit juice intake c. increasing the number of breast-feedings d. discussing the infant's feeding patterns

D. discussing the infant's feeding patterns Rationale: Assessing the current feeding pattern and daily intake is the priority intervention. Talking about solid food consumption may not be appropriate for this infant yet. Discouraging daily fruit juice intake or increasing the number of breastfeedings may not be necessary until the situation is assessed.

When caring for an infant with an upper respiratory tract infection and elevated temperature, an appropriate nursing intervention is to: Give tepid water baths to reduce fever. Give small amounts of favorite fluids frequently to prevent dehydration. Have child wear heavy clothing to prevent chilling. Encourage food intake to maintain caloric needs.

Give small amounts of favorite fluids frequently to prevent dehydration. Preventing dehydration by small frequent feedings is an important intervention in the febrile child. Tepid water baths may induce shivering, which raises temperature. Food should not be forced; it may result in the child vomiting. The febrile child should be dressed in light, loose clothing.

A young child is brought to the emergency department with severe dehydration secondary to acute diarrhea and vomiting. Therapeutic management of this child will begin with: Oral rehydration solution (ORS). Administration of antidiarrheal medication. Clear liquids, 1 to 2 ounces at a time. Intravenous fluids.

Intravenous fluids. Intravenous fluids are initiated in children with severe dehydration. ORS is acceptable therapy if the dehydration is not severe. Diarrhea is not managed by using clear liquids by mouth. These fluids have a high carbohydrate content, low electrolyte content, and high osmolality. Antidiarrheal medications are not recommended for the treatment of acute infectious diarrhea.

Which would be a nutritional goal for a preschool client? Let the child eat only what the child wants. Reduce messiness and spills. Introduce new food gradually and include variety. Eat everything on the plate.

Introduce new food gradually and include variety. Rationale: Mealtimes can become a power struggle between caregivers and the young child. Reassure caregivers that young children go through periods during which they are very particular about food. Therefore, new foods should be gradually introduced and include variety. Foods that look like or smell like other foods they enjoy are the most likely to be eaten. Eating everything on the plate can lead to obesity; the child needs a variety of the 5 food groups; and messiness and spills allow the child to use fine and gross motor skills and practice their use of utensils and cups.

Infants do this gross motor skill at four months

Roll from stomach to back

What is also known as German Measles?

Rubella

4 fine motor development milestones developed at 2 years of age

Stacks object Copy straight lines/circles Throw ball overhand Turns pages

The school nurse observes a child with a peanut allergy in obvious distress, wheezing and cyanotic, after ingestion of some trail mix containing peanuts. Place the interventions the nurse should implement in order of the highest priority to the lowest priority. a. Call Jason's parents and notify them of the situation. b. Call Jason's family practitioner to obtain further orders for medication. c. Promptly administer an intramuscular dose of epinephrine. d. Call 911 and wait for the emergency response personnel to arrive.

c, d, b, a The nurse should first administer epinephrine IM to a child with a food allergy who is in obvious distress, wheezing, and cyanotic. 911 should be called after the epinephrine is administered. The physician should be contacted for further orders and, last, the parents notified of the situation.

Most common infection that causes osteomyelitis

Staph Aureus

The infant weighs 7 lb 4 oz (3,248 g) at birth. If the infant is following a normal pattern of growth, what would be the expected weight for this child at the age of 12 months? 25 lb (11.3 kg) 21 lb 12 oz (9.9 kg) 14 lb 8 oz (6.6 kg) 28 lb 4 oz (12.8 kg)

21 lb 12 oz (9.9 kg) Rationale: The average weight of a newborn is 7.5 lb (3400 g). The infant gains about 30 g each day. By four months of age, the infant has doubled the birthweight. By 1 year of age, the infant has tripled the birthweight and has grown 10 to 12 in (25 to 30 cm). 7.25 lb × 3 = 21.75 lb or 21 lb 12 oz

A 6-month-old infant is admitted with a diagnosis of bacterial meningitis. The nurse would place the infant in which room? A two-bed room in the middle of the hall A private room near the nurses' station A room with an 8-month-old infant with failure to thrive A room with a 12-month-old infant with a urinary tract infection

A private room near the nurses' station Rationale: A child who has the diagnosis of bacterial meningitis will need to be placed in a private room until he or she has received IV antibiotics for 24 hours because the child is considered contagious. Additionally, bacterial meningitis can be quite serious; therefore, the child should be placed near the nurses' station for close monitoring and easier access in case of a crisis.

A child with cystic fibrosis (CF) receives aerosolized bronchodilator medication. When should this medication be administered? Before receiving 100% oxygen After CPT Before chest physiotherapy (CPT) After receiving 100% oxygen

Before chest physiotherapy (CPT) Bronchodilators should be given before CPT to open bronchi and make expectoration easier. Aerosolized bronchodilator medications are not helpful when used after CPT. Oxygen administration is necessary only in acute episodes with caution because of chronic carbon dioxide retention.

Currently the leading cause of death from cancer in children and adolescent from ages 1 to 19

Brain tumors

Most common cranial, facial, defect

Cleft lip/palate

This fine motor skill is usually achieved at 12 months

Feeding self with cup and spoon

Two autoimmune disorders that affect children

Juvenile arthritis SLE

Self-limiting condition in which the blood supply of the head of the femur is disrupted causing vascular necrosis

Legg-Calve-Perthes

This fine motor developmental milestone is usually achieved by 6 months

Transferring objects from one hand to another

When assessing a child for the probable cause of acute bronchiolitis, the nurse focuses on which factor? Viral infections Prenatal complications Environmental allergies Bacterial infections

Viral infections Rationale: Acute bronchiolitis is caused by a viral not bacterial infection. Neither allergies nor prenatal complications contribute to the development of this disorder.

A child with a suspected airway obstruction is brought to the emergency room. The child produces a harsh, strident sound on inspiration (stridor). Where is the obstruction likely to be located based on this information? a. in the larynx b. lower trachea c. bronchioles d. pharynx

a. in the larynx When the vibrations produced as air are forced past obstructions such as mucus in the nose or pharynx, the noise produced is a snoring sound (rhonchi). If the obstruction is at the base of the tongue or in the larynx, a harsher, strident sound on inspiration (stridor) occurs. If an obstruction is in the lower trachea or bronchioles, an expiratory whistle sound (wheezing) occurs.

The nurse is assessing a child with a burn injury for signs of cellulitis. Which finding(s) prompts the nurse to perform interventions for cellulitis? Select all that apply. Koplik spots cold, clammy skin around the infected area chills fever lymphadenopathy

fever lymphadenopathy

An infant has been born and diagnosed with a meningocele. Which action will the nurse incorporate into each contact with this infant? listening for a shrill cry auscultation for bowel sounds careful supine positioning inspection of the cystic sac on the child's back for leakage

inspection of the cystic sac on the child's back for leakage Rationale: Leakage from the cystic area indicates loss of cerebrospinal fluid (CSF) and risk of infection of the central nervous system. Prompt intervention is needed, probably surgical. Listening for bowel sounds confirms intestinal peristalsis but is not necessary with each infant contact. A shrill cry may indicate increased intracranial pressure (ICP). This is important to note yet is not as pressing as being aware of leakage. The baby would be positioned prone, not supine, to protect the sac.

The nurse is reviewing the treatment approaches for a 13-year-old child with Sever disease with the child's parents. What will the nurse include in the teaching? Select all that apply. rest stretching traction nonsteroidal anti-inflammatory drugs (NSAIDs) vancomycin

rest

An 8-year-old child is being treated for tonic-clonic seizures. What should the nurse emphasize when teaching the parents about this disorder? Have the child carry a padded tongue blade with her at all times. Immediately provide medication if a seizure begins. The child should maintain an active lifestyle. Ensure quiet time late in the day, when seizure activity is most likely to occur.

The child should maintain an active lifestyle. Rationale: As a rule, children with seizures should attend regular school and participate in physical education classes and active sports. Antiseizure medication is ineffective during a seizure because most medication needs to achieve a therapeutic level to be effective. Padded tongue blades are not used in people with a seizure disorder. There is no specific time of day when a seizure can occur.

The nurse is providing teaching to the parents of a child with varicella. Which statement indicates that the parents have understood the instructions? "We need to make sure that our child washes the hands frequently." "If our child has a fever, we can give our child some aspirin." "We should apply alcohol to the lesions every four hours." "The lesions should eventually form soft crusts that drain."

We need to make sure that our child washes the hands frequently." Rationale: The child with varicella needs to wash the hands frequently with antibacterial soap to reduce bacterial colonization. A cool bath with soothing colloidal oatmeal may help the skin discomfort. Alcohol would be too drying to the skin. Acetaminophen, not aspirin, should be used to reduce fever. The lesions should eventually crust over. Soft crusts with drainage may suggest an infection.

An otherwise healthy 18-month-old child with a history of febrile seizures is in the well-child clinic. Which statement by the father would indicate to the nurse that additional teaching should be done? "The most likely time for a seizure is when the fever is rising." "My child will likely outgrow these seizures by age 5." "I always keep phenobarbital with me in case of a fever." "I have ibuprofen available in case it's needed."

"I always keep phenobarbital with me in case of a fever." Rationale: Anticonvulsants, such as phenobarbital, are administered to children with prolonged seizures or neurologic abnormalities. Ibuprofen, not phenobarbital, is given for fever. Febrile seizures usually occur after age 6 months and are unusual after age 5. Treatment is to decrease the temperature because seizures occur as the temperature rises.

After teaching a group of nursing students about developmental milestones for children, between ages 1 and 4 years, the instructor determines that the teaching was successful when the students identify which of the following as a gross motor developmental milestone that occurs between 2 to 3 years of age? A. jumping in place B. riding a tricycle C. climbing D. standing on one foot

A. jumping in place Rationale: A gross motor developmental milestone for a 2 to 3 year-old child includes jumping in place. Riding a tricycle occurs at 3-4 years of age. Climbing occurs at 18 months to 2 years. At 12-18 months, the child can stand on one foot with help.

The parents of twin 2-year-old children are asking the nurse how to discipline the children. It seems they feed off one another's feelings and many times get into fights over everything. When giving advice about discipline, which statement should be shared with the parents? Select all that apply. A. The rule with time out is tell the child why they are going to time out and keep it to 1 minute per year of age. B. Warnings about going to time out should not be done. If the child breaks a rule, send them directly to time out. C. The discipline the parents choose should be consistent for every time the child breaks the same rule. D. If possible, try to praise correct behavior rather than punish wrong behavior. E. It is normal for children to hit their friends if the friend takes one of their toys, so the friend should be warned not to take toys that are being played with.

A. tell the child why they are going to time out and keep it to 1 minute per year of age. C. Discipline should be consistent every time D. praise correct behavior rather than punish wrong behavior. Rationale: Parents should begin to instill some sense of discipline early in life because part of it involves setting safety limits and protecting others or property. Parents need to be consistent with their punishment. Rules are learned best if correct behavior is praised rather than wrong behavior punished. A "time-out" is a technique to help children learn that actions have consequences. To use a time-out effectively, parents first need to be certain their child understands the rule they are trying to enforce (e.g., "You can't hit people. If you hit your friend, you'll have time-out"). Parents should give one warning. If the child repeats the behavior, parents select an area that is nonstimulating, such as a corner of a room or a hallway. A guide as to how long children should remain in their time-out chair is 1 minute per year of age (e.g., a 2-year-old would stay in the corner for 2 minutes).

A 13-year-old child has had a near-drowning experience. The nurse notices the child has labored breathing and a cough. Which action is the nurse's priority? Have the child sit up straight in a chair. Administer 100% oxygen by mask. Check the child's capillary refill time. Provide sedation as ordered.

Administer 100% oxygen by mask. Rationale: Management of the near-drowning victim focuses on assessing the ABCs and correcting hypoxemia. Administering oxygen is the primary intervention to assist breathing. It is best to let the child assume his own most comfortable position. Checking capillary refill time helps determine ineffective tissue perfusion. Providing sedation is an intervention for pain that will be assessed after effective breathing is established.

The nurse is caring for a child and notes periorbital edema on the left eye with urticaria. Which action by the nurse is priority? Ask if the child has allergies. Assess lung sounds bilaterally. Administer a corticosteroid. Evaluate fluid volume status.

Assess lung sounds bilaterally. Rationale: When a child has signs of angioedema, the nurse's priority is to ensure the airway is patent, by assessing breathing sounds, because angioedema can cause laryngeal obstruction and asphyxiation. Evaluating fluid volume status, asking about allergies, and administering a corticosteroid are all actions that could be performed after first ensuring the child was breathing.

An infant's parents ask the nurse about preventing otitis media (OM). What should the nurse recommend? Avoid tobacco smoke. Avoid children with OM. Bottle-feed or breastfeed in supine position. Use nasal decongestant.

Avoid tobacco smoke. Eliminating tobacco smoke from the child's environment is essential for preventing OM and other common childhood illnesses. Nasal decongestants are not useful in preventing OM. Children with uncomplicated OM are not contagious unless they show other upper respiratory infection symptoms. Children should be fed in an upright position to prevent OM.

The nurse is reviewing the health history and physical examination of a child diagnosed with heart failure. What would the nurse expect to find? Select all that apply. Crackles on lung auscultation Hypertension Tiring easily when eating Bradycardia Shortness of breath when playing

Crackles on lung auscultation Shortness of breath when playing Tiring easily when eating Rationale: Manifestations of heart failure include difficulty feeding or eating or becoming tired easily when feeding or eating, shortness of breath with exercise intolerance, crackles and wheezes on lung auscultation, tachycardia, and hypotension.

An 18-month-old child is admitted with signs of increased intracranial pressure. What should the nurse observe when assessing this patient? Increased temperature and decreased respiratory rate Numbness of fingers and decreased temperature Decreased level of consciousness and increased respiratory rate Increased pulse rate and decreased blood pressure

Increased temperature and decreased respiratory rate Rationale: Manifestations of increased intracranial pressure include increased body temperature and decreased respiratory rate. Pulse rate slows, and the blood pressure increases.

The nurse is teaching the parents of an infant with a cleft palate about strategies for feeding. What should the nurse include in the teaching? Select all that apply. Use a squeezable bottle. Keep the head upright at 45 degrees. Use a regular nipple for bottle feeding. Provide a referral to an orthodontist. Review feeding devices.

Keep the head upright at 45 degrees. Provide a referral to an orthodontist. Review feeding devices.

A nurse is reading a journal article about adolescents and major causes of injuries in this age group. The nurse demonstrates understanding of this information by identifying which situation as the major cause of adolescent injuries? Drowning Motor vehicle crashes Violence Suicide

Motor vehicle crashes Rationale: Although drowning, violence, and suicide are causes of adolescent injury, the largest number of adolescent injuries are due to motor vehicle crashes.

The nurse is assessing the parents interacting with their infant. Which of the following would indicate to the nurse that attachment is occurring? Select all that apply. Parents refrain from inspecting the baby's body. Parents hold the baby close to the body. Parents talk to the baby while holding the baby Parents avoid snuggling with the baby. Parents make eye-to-eye contact.

Parents hold the baby close to the body. Parents talk to the baby while holding the baby Parents make eye-to-eye contact. Rationale: Attachment is the emotional bond that creates an important foundation for the relationship between the parent and the infant. Certain identifiable behaviors—eye-to-eye contact, physical contact, and communication—indicate that attachment is occurring.

A child is refusing to use the potty and having accidents, even though he has achieved toilet training. This is an example of which type of behavior? Regression Positive redirection Desensitization Phobia

Regression Rationale: Regression is a change from present behaviors to past developmental levels of behavior. Positive redirection is verbally guiding the child toward the accepted behavior. Desensitization occurs when the fear is conquered by approaching it little by little. If the fear becomes a phobia, then the health care team must be consulted.

When describing play by the school-aged child to a group of nursing students, the instructor would emphasize the need for which of the following? rules recreation physical activity ritualism

Rules Rationale: Middle childhood is characterized by play that embodies the need for rules and structure. Although recreation and physical activity may be involved, rules and structure are essential for play in this age group.

A child with Addison disease has been admitted with a history of nausea and vomiting for the past 3 days. The client is receiving IV glucocorticoids. Which intervention would the nurse implement? Measure intake and output. Monitor sodium and potassium levels. Weigh daily. Take glucometer readings as ordered.

Take glucometer readings as ordered. Rationale: IV glucocorticoids raise the glucose levels and often require coverage with insulin. Measuring the intake and output at this time is not necessary. Sodium and potassium would be monitored when the client is receiving mineralocorticoids. Daily weights are not necessary at this time.

The nurse is caring for a 3-month-old infant undergoing treatment with a Pavlik harness for developmental dysplasia of the hip (DDH). Which finding requires further follow up by the nurse? The parents report checking the skin for areas of redness, irritation, and breakdown. The harness is being worn underneath the clothes. The diaper is fastened underneath the straps. The child remains in the harness 23 hours per day.

The harness is being worn underneath the clothes. Rationale: The nurse should follow up on the harness being worn underneath the child's clothes. The child should wear a t-shirt or similar garment under the harness to reduce skin irritation. Checking the skin for areas of redness, irritation, and breakdown; having the child wear the harness for 23 hours per day; and fastening the diaper underneath the straps of the harness are all appropriate steps in the care of a child using a Pavlik harness and do not require further follow up by the nurse.

An adolescent comes into the emergency department with a foot wound. Upon assessment, the nurse learns that the patient is a runaway and has been living on the streets. Which is the most appropriate care for the nurse to provide to the client at this time? Discuss the importance of a diet high in protein and vitamin C. Treat the wound and provide wound care supplies. Recommend returning to live with parents. Explain how the wound needs to be flushed with water every 4 hours.

Treat the wound and provide wound care supplies. Rationale: The nurse is not aware of the adolescent runaway's family situation so suggesting returning home with parents may or may not be appropriate. Because the adolescent runaway has no money for any kind of food, giving instructions to eat foods high in protein and vitamin C makes no sense. If the adolescent runaway does not have a source of running water, telling them to flush the wound with water every 4 hours will be impossible. The best care at this time would be for the nurse to treat the adolescent runaway's wound and provide him or her with wound care supplies.

A child has been admitted to the inpatient unit to rule out acute Kawasaki disease. A series of laboratory tests have been ordered. Which findings are consistent with this disease? Select all that apply. negative C-reactive protein levels elevated erythrocyte sedimentation rate (ESR) reduced hemoglobin levels reduced white blood cell count reduced platelet levels

elevated erythrocyte sedimentation rate (ESR) reduced hemoglobin levels Rationale: Kawasaki disease is an acute systemic vasculitis occurring mostly in children 6 months to 5 years of age. It is the leading cause of acquired heart disease among children. The CBC count may reveal mild to moderate anemia, an elevated white blood cell count during the acute phase, and significant thrombocytosis (elevated platelet count [500,000 to 1 million]) in the later phase. The erythrocyte sedimentation rate (ESR) and the C-reactive protein (CRP) level are elevated.

The nurse is caring for a child with a gastrointestinal disorder and measuring intake and output. The nurse observes that the child is demonstrating symptoms of adequate hydration when the child has which findings? Select all that apply. loose stools fontanels (fontanelles) with normal tension oral intake pink and moist mucous membranes adequate skin turgor

fontanels (fontanelles) with normal tension pink and moist mucous membranes adequate skin turgor

A 6-month-old girl weighs 14.7 pounds (6.7 kg) during a scheduled check-up. Her birth weight was 8 pounds (3.6 kg). What is the priority nursing intervention? Discussing the child's feeding patterns Talking about solid food consumption Discouraging daily fruit juice intake Increasing the number of breast-feedings

Discussing the child's feeding patterns Rationale: Assessing the current feeding pattern and daily intake is the priority intervention. Talking about solid food consumption may not be appropriate for this child yet. Discouraging daily fruit juice intake or increasing the number of breastfeedings may not be necessary until the situation is assessed.

The nurse is caring for a chronically ill adolescent client. What can the nurse do to maintain stimulation and support the client's sense of identity while hospitalized? Plan activities around scheduled rest periods. Explain food choices appropriate to the prescribed diet. Teach the name and indications for use of all medications. Encourage communicating with friends through social media.

Encourage communicating with friends through social media. Rationale: To encourage stimulation while supporting the adolescent client's sense of identity while hospitalized, the nurse should encourage the client to communicate with friends through social media. Planning activities around rest periods does not promote stimulation. Explaining food choices does not promote stimulation. Learning about medications does not promote stimulation.

The nurse is caring for a child with history of asthma who presents to the emergency department with wheezing, tachypnea, and dyspnea. What will the nurse do first? a. Ask what may have triggered the attack. b. Place the child in high-Fowler position. c. Assess the child's pulse oximetry reading. d. Apply oxygen via nasal cannula at 2 liters.

b. Place the child in high-Fowler position. Rationale: The nurse will first elevate the head of the bed to improve the child's ability to breathe. Elevating the head of the bed allows the diaphragm to expand, consequently maximizing ventilation and oxygenation. After elevating the head of bed, the nurse will assess the pulse oximetry and apply oxygen if needed. After stabilizing the child, the nurse can ask what may have triggered the asthma attack.

The nurse is providing anticipatory guidance to the parent of a 9-month-old infant during a well-baby visit. Which topic would be most appropriate? A.Advising how to create a toddler-safe home B. Warning about leaving small objects on the floor C. Cautioning about putting the infant in a walker D. Instructing on safety procedures during baths

A.Advising how to create a toddler-safe home Rationale: The most appropriate topic for this parent would be advising on how to create a toddler-safe home. The infant will very soon be pulling oneself up to standing and cruising the house. This will give the infant access to areas yet unexplored. Warning about leaving small objects on the floor, telling about safety procedures during baths, and cautioning about using baby walkers would no longer be anticipatory guidance as the infant has passed these stages.

The nurse has received morning report on a group of pediatric clients. Which pediatric client will the nurse see first? a. an infant with rhinorrhea, coughing, and oxygen saturation of 92% b. a toddler with a temperature of 100.1°F (38°C), and a harsh, barking cough c. a preschool child with crackles in the right lower lobe and chest pain d. a school-age child with dysphagia, drooling, and a hoarse voice

Answer: D Rationale: The child with signs and symptoms of epiglottitis should be seen first because epiglottitis is an emergency that can quickly cause airway obstruction. A child with signs of bronchiolitis with an oxygen saturation of 92% is more stable than this child with epiglottitis, as is the toddler with signs of croup and the child with signs and symptoms of pneumonia.

The newborn weighs 7 lb 4 oz (3,300 g) at birth. If the newborn is following a normal pattern of growth, what would be the expected weight for this newborn at the age of 12 months? a. 14 lb 8 oz (6.6 kg) b. 21 lb 12 oz (9.9 kg) c. 25 lb (11.3 kg) d. 28 lb 4 oz (12.8 kg)

B. 21 lb 12 oz (9.9 kg) Rationale: The average weight of a newborn is 7.5 lb (3400 g). The newborn gains about 30 g each day. By 1 year of age, the newborn has tripled the birth weight and has grown 10 to 12 in (25 to 30 cm). 7.25 lb × 3 = 21.75 lb or 21 lb 12 oz (9.9 kg)

The nurse is caring for an 8-year-old child hospitalized for the management of cystic fibrosis. How will the nurse support the child's growth and development? Select all that apply. Provide explanations of treatments. If caregivers leave, provide the expected time of return. Encourage the child to keep up with schoolwork. Encourage communication with friends. Explain to parents that regression is likely to occur.

Encourage the child to keep up with schoolwork. Encourage communication with friends.

A 9-year-old child has just been diagnosed with Graves disease. Which symptom should the nurse expect in this child? Select all that apply. Moist skin Exophthalmos (protruding eyes) Increased basal metabolic rate Obesity Nervousness Lethargy

Moist skin Exophthalmos (protruding eyes) Increased basal metabolic rate Nervousness Rationale: In Graves disease, children gradually experience nervousness, tremors, loss of muscle strength, and easy fatigue. Their basal metabolic rate, blood pressure, and pulse all increase. Their skin feels moist and they perspire freely. An exophthalmos-producing pituitary substance causes the prominent-appearing eyes that accompany hyperthyroidism in some children. Obesity and lethargy are symptoms of hypothyroidism, not of Graves disease (hyperthyroidism).

The mother of a 3-month-old baby is concerned because the child is not able to sit independently. How should the nurse respond to this mother's concern? Sitting ability and the age of first tooth eruption are correlated. Most babies sit steadily at 3 months. Most babies sit steadily at 4 months. Most babies do not sit steadily until 8 months.

Most babies do not sit steadily until 8 months. Rationale: An 8-month-old child can sit securely without any additional support. Babies are not able to sit steadily at age 3 or 4 months. Sitting ability does not correspond with tooth eruption.

The nurse is performing Adam's forward-bending test on a child for scoliosis screening. What action will the nurse include in the screening? Note if one side of the back is higher than the other. Note whether the angle of the spine is greater than 20 degrees. Examine the child from the side. Have the child bend forward at a 45-degree angle.

Note if one side of the back is higher than the other. Rationale: The nurse will note if one side of the back is higher than the other while performing Adam's forward-bending test. The child should bend forward until the spine is horizontal during this test, not at a 45-degree angle. The nurse should examine the child from the rear. Rather than assessing for a specific angle measurement, the nurse should evaluate whether or not the child's back appears straight.

What will be the nurse's next action after noting dimpling and a tuft of hair located in the lumbosacral area of the preschool child during examination? Record and refer the finding for follow-up to the pediatrician. Move on to other assessments without calling attention to the difference. Snip the tuft of hair off close to the skin for hygienic reasons. Inspect for precocious hair growth in the genital and underarm areas.

Record and refer the finding for follow-up to the pediatrician. Rationale: Dimpling and hair growth may signal spina bifida occulta, which usually is benign. However, some complications can be associated, and further investigation is warranted to prevent possible damage to the spinal cord. Magnetic resonance imaging (MRI) is often the diagnostic tool used. No hygienic concerns need prevail. These findings do not suggest development of precocious puberty or any other hormonal problem. The dimpling and hair tuft must be clearly explained to the parents.

Gross motor milestones achieved by 3 years of age

Walk up and down stairs with one foot on each step jumping forward

The parents of a 2 year old are concerned because the toddler only says a few words. What strategies should the nurse suggest to the parents? Select all that apply. Use baby talk when speaking. Have the toddler watch educational television. Name aloud the objects being played with. Always answer questions using correct grammar. Use pronouns when speaking. Read books aloud to the toddler.

- Read books aloud to the toddler. - Name aloud the objects being played with. - Always answer questions using correct grammar.

The nurse is teaching the parents of a child newly diagnosed with trisomy 21. What will the nurse include in the teaching? Select all that apply. infertility in the child methods to communicate with the child in developmentally appropriate ways community resources key support organization websites importance of dietary restrictions to prevent the condition from worsening

-methods to communicate with the child in developmentally appropriate ways -community resources -key support organization websites Rationale: The nurse will educate the parents regarding community resources, key support organization websites, and methods to communicate with the child that are developmentally appropriate. Counseling the parents about infertility in the child applies to Turner syndrome, not trisomy 21. The importance of dietary restrictions for preventing the condition from worsening is applicable to phenylketonuria (PKU), not trisomy 21.

The nurse is caring for a child who is critically ill and requiring fluid resuscitation. Which intravenous fluids are appropriate for use? Select all that apply. normal saline 5% lactated Ringer 10% dextrose in water 5% dextrose in water lactated Ringer

-normal saline -lactated Ringer Rationale: If the circulation or perfusion is compromised, then fluid resuscitation is necessary. Establish large-bore intravenous (IV) access immediately and administer isotonic fluid rapidly. Lactated Ringer solution and normal saline are isotonic solutions. 5% and 10% dextrose in water and 5% lactated Ringer solutions are hypertonic and not appropriate for use in this scenario.

A nurse is preparing a presentation for a parent group about media and school-aged children. Which of the following would the nurse most likely include in the presentation? Select all that apply. A. Media use is displacing other activities in a child's life. B. Children watch about 2 hours of television on average per day. C. Programs viewed often glamorize violence. D. A child's weight decreases with the number of hours spent watching television. E. Risky behaviors are often portrayed with little emphasis on consequences.

A,C,E Rationale: Media use by children and adolescents has increasingly displaced other activities in the child's life, such as reading, exercising, and playing with friends. The average child or adolescent watches an average of 3 to 4 hours of television a day. Some of the major concerns associated with media include the glamorization of violence, increased weight associated with the number of hours spent watching television, and the promotion of risky behaviors with little emphasis on consequences.

The parents of an 8-month-old infant voice concern to the nurse that their infant is not developing motor skills as the infant should. What question would be appropriate for the nurse to ask in determining if their fears are warranted? A. "Does the infant move a toy back and forth from one hand to the other?" B. "Does the infant place toys into a box or container and take them out?" C. "Is the infant able to drink with a cup by oneself?" D. "Is the infant able to hold a pencil and scribble on paper?"

A. "Does the infant move a toy back and forth from one hand to the other?" Rationale: Transferring an object from one hand to the other is expected at 7 months of age, so this would be expected of an 8-month-old infant. The other options are not expected until later months, so questioning the parents about these skills would not help in determining if the infant was at the motor skill developmental level that should be expected.

The nurse is assessing a 5-year-old child. Which statement by the parents would indicate the child is behind in communication skills? A. "My child knows the city we live in, but not the street address." B. "My child can describe how a microwave and television remote work." C. "My child talks about things that happened the previous week and even future plans." D. "My child finally has speech that my family and friends can understand."

A. "My child knows the city we live in, but not the street address." Rationale: Normal communication skills in a 5-year-old child include the ability to state one's name and address, so this statement would indicate a delay. The other comments (recalling past and future plans, understanding how things work and has speech that most people can understand clearly) indicate a normal communication skill for a 5-year-old child.

The parent of a 4-year-old child has expressed concern that the child is wetting the bed several times each week. What should the nurse teach the parent? A. Bed-wetting is not an unexpected behavior at this age. B. The child's fluid intake should be limited after 3:00 p.m. C. The parents should try to dialogue with the child about possible causes and solutions. D. The parents should provide incentives for keeping the bed dry.

A. Bed-wetting is not an unexpected behavior at this age. Rationale: Bed-wetting is not normally considered a problem unless it persists after the ages of 5 to 7. Limiting fluids after midafternoon may create a risk for dehydration.

A child has a nasogastric (NG) tube to continuous low intermittent suction. The physician's prescription is to replace the previous 4-hour NG output with a normal saline piggyback over a 2-hour period. The NG output for the previous 4 hours totaled 50 mL. What milliliter/hour rate should the nurse administer to replace with a normal saline piggyback? _____ Record your answer as a whole number.

25

A child should have all 20 baby teeth by _______ years of age.

3 years old

Which nursing intervention is priority when caring for a child with HIV? Administer prescribed medications. Assess pain after invasive procedures. Assist the child with daily activities. Review laboratory CD4 counts daily.

Administer prescribed medications. Rationale: Although assisting with activities, assessing pain, and reviewing CD4 counts are all important, the priority when caring for a child with HIV is to administer prescribed medications. Prescribed medications prevent progressive deterioration of the immune system and provide prophylaxis against opportunistic infections.

The nurse is caring for a newborn diagnosed with an inborn error of metabolism with several referrals ordered. What referral would the nurse place as the priority for the newborn? dietitian spiritual advisor genetic counseling community support group

dietitian Rationale: The newborn with an inborn error of metabolism will have specific dietary guidelines, and the parents need to understand the dietary restrictions soon after birth to ensure the newborn is not harmed. The other referrals are important and should be addressed soon after birth.

The nurse is caring for a 2-month-old with a cleft palate. The child will undergo corrective surgery at age 3 months. The mother would like to continue breastfeeding the baby after surgery and wonders if it is possible. How should the nurse respond? "There is a good chance that you will be able to breastfeed almost immediately." "Breastfeeding is likely to be possible, but check with the surgeon." "After the suture line heals, breastfeeding can resume." "We will have to wait and see what happens after the surgery."

"Breastfeeding is likely to be possible, but check with the surgeon." Rationale: Postoperatively, some surgeons allow breastfeeding to be resumed almost immediately. However, the nurse needs to advise the mother to check with the surgeon to determine when breastfeeding can resume. Telling the mother that she has to wait until the suture line heals may be inaccurate. Telling her to wait and see does not answer her question.

A 2-year-old child and the parents are at the office for a follow-up visit. The child has had excessive hormone levels in the recent blood work, and the parents question why this was not found sooner. Which response by the nurse would be most appropriate? "Have there been signs and symptoms that you should have reported to the health care provider?" "It takes time to determine the level of functioning of endocrine glands." "As endocrine functions become more stable throughout childhood, alterations become more apparent." "Endocrine disorders are hard to detect and you are lucky that we have found it when we did."

"As endocrine functions become more stable throughout childhood, alterations become more apparent." Rationale: The endocrine glands are all present at birth; however, endocrine functions are immature. As these functions mature and become stabilized during the childhood years, alterations in endocrine function become more apparent. Thus, endocrine disorders may arise at any time during childhood development.

The nurse is caring for a 8-year-old child. During a routine wellness examination, the parent tells the nurse that the spouse has enrolled the child in a mini pom-pom cheering squad. The child dislikes it immensely, but the spouse does not want the child to be a "quitter." The parent asks for some guidance. How should the nurse respond? "Bad experiences can cause the child to avoid other similar activities." "Tell your spouse that requiring the child to continue is inappropriate." "Your child may not be able to keep up with the instructions." "This may not suit your child's temperament or physical abilities."

"Bad experiences can cause the child to avoid other similar activities." The nurse must provide the mother with concrete information that will help the father understand that allowing the girl to quit an activity she dislikes will not make her a "quitter." The father is most likely to respond to the fact that this may cause lasting repercussions. Telling the husband that his expectations are inappropriate does not teach. Not being able to keep up with instructions or not suiting the child's temperament or physical abilities may be true, but these responses are not therapeutic or helpful in teaching the parents.

A nurse is teaching the parents of an infant diagnosed with phenylketonuria (PKU) about dietary restrictions. The parents ask the nurse what will happen if their infant does not follow the diet. How should the nurse respond? "Not following the diet could cause changes in your infant's facial structure." "Hearing or vision impairments can occur with diet noncompliance." "Dietary noncompliance can cause your infant to have poor muscle tone." "Developmental delays can result from not adhering to the prescribed diet."

"Developmental delays can result from not adhering to the prescribed diet." Rationale: Not adhering to dietary restrictions can cause developmental delays and intellectual impairment in a client with phenylketonuria (PKU). Hearing and vision impairments and poor muscle tone are associated with Down syndrome, not PKU. Changes in facial structure such as micrognathia (small jaw and mouth) is a manifestation of trisomy 13, not PKU.

An infant with poor feeding is suspected of having a congenital heart defect. The parents are asking why a chest x-ray is necessary in their infant. What is the best response from the nurse? "It will show if blood is being shunted." "This image will clarify the structures within the heart." "It will determine if the heart is enlarged." "It will determine disturbances in heart conduction."

"It will determine if the heart is enlarged." Rationale: Chest x-rays are performed to see if the heart is enlarged. This will determine if the heart muscle is increasing in size. Disturbances in heart conduction are detected by an EKG. Visualizing where blood is being shunted is through the echocardiogram. The image used to clarify the structures of the heart is the MRI.

The nurse is providing education to the parents of a child with hydrocephalus who has just had a shunt inserted. When discussing the child's condition with the parents, which would be most appropriate? "Always keep the child's head raised 30 degrees." "Be sure to call the doctor if your child gets a persistent headache." "Her autoregulation mechanism to absorb spinal fluid has failed." "Tell me your concerns about your child's shunt."

"Tell me your concerns about your child's shunt." Rationale: Always start by assessing the family's knowledge. Ask them what they feel they need to know. Knowing when to call the doctor and how to raise the child's head are important, but they might not be listening if they have another question on their minds. "Autoregulation" is too technical. Base information on the parents' level of understanding.

The nurse is assessing a 6-month-old child. The mother asks when the soft area in her child's head will go away. What is the best response by the nurse? "Soft spots on the child's head should have closed by now." "The soft spots may stay open until your child is two or three years old." "The anterior fontanel typically closes 12 to 18 months of age." "The area is called a fontanel. They remain open to allow for rapid brain growth in the first months of life."

"The anterior fontanel typically closes 12 to 18 months of age." Rationale: The anterior fontanel typically closes by the age of 9 to 18 months. Fontanels are soft areas on the skull that remain open in infancy to allow for rapid brain growth in the first months of life. This answer is a true statement but does not answer the mother's question.

A parent calls the hospital nursing hotline and asks, "My 8-week-old infant cries 8 hours a day, and is hard to console. Is that normal?" What should the nurse's response be to this parent? "Let me ask you some more questions to help me answer your question." "Yes, many infants cry all the time at that age." "No, this is not normal, please call your health care provider." "Yes, how is your infant sleeping, maybe your infant is just tired."

Answer: "Let me ask you some more questions to help me answer your question.." Rationale: The nurse should seek more information to assess the infant's symptoms. The symptoms suggest colic, which is characteristic of an infant who cries more than 3 hours a day and is fussy and hard to console. The other responses are non-therapeutic and do not seek further information to gather a history.

The nurse is caring for a 1-day-old newborn with patent ductus arteriosus with the following vital signs: pulse 160, respirations 80, oxygen saturation 92%, retractions, and crackles noted in bilateral lungs. Which nursing actions are appropriate at this time? Select all that apply. a. Begin indomethacin infusion. b. Apply oxygen via oxyhood. c. Administer furosemide. d. Initiate intravenous access. e. Feed a high-calorie formula.

A, B, C, D - Administer furosemide. - Initiate intravenous access. - Apply oxygen via oxyhood. - Begin indomethacin infusion. Rationale: When a newborn with patent ductus arteriosus shows signs of significant blood flow to lungs (retractions, crackles, tachypnea, and hypoxia), nursing actions will focus on applying oxygen to improve oxygenation and decrease work of breathing. Nursing interventions also include reducing cardiac workload and pulmonary flow by initiating intravenous access to administer a diuretic to reduce extra fluid and indomethacin to cause closure of the patent ductus artery and stop increased pulmonary blood flow. Feeding the infant is not a priority at this time as aspiration may result from the inability to coordinate sucking and swallowing with increased work of breathing.

A 7-year-old child shows symptoms of anaphylactic shock. Which would be appropriate for the nurse to do immediately? Select all that apply. Administer epinephrine as ordered. Initiate intravenous access. Increase fluid intake. Administer oxygen. Teach the child how to use an epinephrine autoinjector.

Administer epinephrine as ordered. Initiate intravenous access. Administer oxygen. Rationale: Immediate management for anaphylaxis includes administering epinephrine intramuscularly or intravenously to reverse histamine release and hypotension, rapidly assessing the child's ABCs and intervening as necessary, maintaining a patent airway, administering oxygen, starting an IV, stopping the antigen release or slowing its absorption, administering albuterol aerosols to reverse bronchospasm, and administering antihistamines to counteract histamine release. Fluids would be administered if hypotension does not respond to epinephrine and antihistamines. Teaching the child how to use an epinephrine autoinjector would be done later, once the child's condition is stabilized.

What should the nurse stress in a teaching plan for the mother of an 11-year-old boy with ulcerative colitis? Coping with stress and avoiding triggers Teaching daily use of enemas Preventing the spread of illness to others Nutritional guidance and preventing constipation

Coping with stress and avoiding triggers Coping with the stress of chronic illness and the clinical manifestations associated with ulcerative colitis (diarrhea, pain) are important teaching foci. Avoidance of triggers can help minimize the impact of the disease and its effect on the child. Ulcerative colitis is not infectious. Although nutritional guidance is a priority teaching focus, diarrhea is a problem with ulcerative colitis, not constipation. Daily enemas are not part of the therapeutic plan of care.

A school health nurse is supporting a 15-year-old adolescent with acne. What is a common myth related to acne in adolescent populations? Diet plays a significant role in acne production. Do not pick or squeeze acne lesions because it will just increase symptoms. Excessive face washing is not necessary to prevent lesions from forming. Makeup may increase lesion formation.

Diet plays a significant role in acne production. Rationale: Diet does not actually influence the development of acne lesions; rather, acne is caused by the changes in puberty, specifically the rapid increase in androgen secretion, which causes the sebaceous glands to become active.

The nurse caring for a toddler immediately after a fall from a grocery cart will avoid moving which body area as the child is examined? Torso Head and neck Clavicle Lower extremities

Head and neck Rationale: The head and neck should remain immobilized until cervical spine injury is ruled out. Motion in this area could damage the spinal cord. The rest of the child's body should be examined carefully so as not to aggravate an unsuspected injury. The clavicle is the bone most frequently fractured during childhood.

What should be included in the teaching plan for a child with type 1 diabetes who is going home on insulin therapy? It is absolutely normal for the growing child to require an increase in insulin; this does not mean his/her condition is getting worse. Once glucose control is established, there will never be a need for an increase in the amount of insulin administered. Children show an increased need for insulin during the first months after glucose control is established. All children should be on at least two types of insulin to establish glucose control.

It is absolutely normal for the growing child to require an increase in insulin; this does not mean his/her condition is getting worse. Rationale: Children show a decreased need for insulin shortly after glucose control has been established, which is referred to by some as the "honeymoon phase" and should be described to parents so that they do not get any false hope that the child does not need insulin. As children grow, they will require increased doses of insulin to maintain glucose control, and not all children need to receive two types of insulin. Insulin treatment should be based on each individual child.

During the routine physical assessment of a 2-year-old, the nurse identifies which finding as being abnormal for this age group? blood pressure of 120/80 mm Hg wears diapers since not potty trained respiration rate of 20 bpm, abdominal breathing noted heart rate of 92 beats/min, regula

blood pressure of 120/80 mm Hg Rationale: Normal assessment findings of toddlers include: respirations slow slightly but continue to be mainly abdominal; heart rate slows from 110 to 90 beats/min; blood pressure increases to about 99/64 mm Hg; and not potty trained until complete myelination of the spinal cord has occurred. The only abnormal finding is an elevated blood pressure. However, this can occur if the toddler is crying or upset about the assessment.

Linda, a 14-year-old, and her mother are in the office. As Linda goes to the bathroom, her mother stops the nurse and asks about the changes that Linda is going through. She would like to talk to her about sex and the changes but she is unsure of how to do this. As the nurse, which of the following would be appropriate for you to suggest? Do not initiate any conversation; let the teen come and seek you out for any advice and answers. Promote open lines of communication, encourage listening, don't lecture, and share family values. Encourage her to talk to her peers and teachers in health class about any concerns that she has. Discuss with the adolescent the experiences that you had so that she can connect on a personal level.

Promote open lines of communication, encourage listening, don't lecture, and share family values. Rationale: Due to the influence of television and movies, many teenagers have an idealized body image or misinformation about sex. In many case the teenager receives excellent information in health education classes. There still may be many unanswered questions it is important for the parents to share family values regarding sexual behavior. By promoting open lines of communication between parents and teens, the teenager can go to the parents with problems or questions. Encourage parents to listen to their teens than lecture them.

The nurse is caring for an infant with suspected pyloric stenosis. Which clinical manifestation would indicate pyloric stenosis? Rounded abdomen and hypoactive bowel sounds Visible peristalsis and weight loss Abdominal rigidity and pain on palpation Distention of lower abdomen and constipation

Visible peristalsis and weight loss Visible gastric peristaltic waves that move from left to right across the epigastrium are observed in pyloric stenosis, as is weight loss. Abdominal rigidity and pain on palpation, and rounded abdomen and hypoactive bowel sounds, are usually not present. The upper abdomen is distended, not the lower abdomen.

Other than providing direct care to children, what is the major role of nurses in the care of nearly all children with neuromuscular disorders? consoling parents teaching children self-care helping with specialized equipment coordinating care by specialists

coordinating care by specialists Rationale: Being part of a multidisciplinary team and coordinating the care the child usually needs from a variety of specialists is an essential and major role. The other nursing activities are important as well, but many children/families require individual interventions.

A 3-year-old child is admitted to the hospital with osteomyelitis of the right femur. The nurse would expect to start an IV and antibiotic after blood is drawn for which lab test? platelets white blood cell count hemoglobin and hematocrit culture

culture Rationale: Only the culture will indicate which antibiotic is the correct medication to give for the infection.

A nurse is obtaining the history from a parent of a child who experiences absence seizures. Which would the nurse expect the parent to describe? loss of motor activity accompanied by a blank stare loss of muscle tone and loss of consciousness sudden, brief jerking motions of a muscle group brief, sudden onset with muscles that become tense

loss of motor activity accompanied by a blank stare Rationale: An absence seizure consists of a sudden, brief arrest of the child's motor activity accompanied by a blank stare and loss of awareness. A tonic seizure consists of a brief onset of increased tone. A myoclonic seizure is characterized by sudden, brief jerks of muscle groups. An atonic seizure involves a sudden loss of muscle tone and loss of consciousness.

During a complete physical assessment of a preadolescent boy, the nurse correctly recognizes which finding as being the first change of puberty? increase in height deepening voice development of axillary hair testicular enlargement

testicular enlargement Rationale: The first sign of pubertal changes in boys is testicular enlargement in response to testosterone secretion, usually occurring in Tanner stage 2. As testosterone levels increase, the penis and scrotum enlarge, hair distribution increases, and scrotal skin texture changes.

The nurse is caring for a child who is about to undergo surgical correction of scoliosis. What will the nurse include in the child's postoperative care? Select all that apply. Ask if the child has numbness or tingling. Begin encouraging the child to walk on postoperative day 3. Assess the ability to bend knees and raise legs. Reposition every 2 hours using the logrolling technique. Have the child sit up for meals on postoperative day 2.

Ask if the child has numbness or tingling. Begin encouraging the child to walk on postoperative day 3. Assess the ability to bend knees and raise legs. Reposition every 2 hours using the logrolling technique.

When assessing a child and parents during a well-child visit, the nurse determines that the child is experiencing night terrors. Which would the nurse be most likely to suggest to assist the parents in dealing with this issue? A. allowing the episode to take its course B. waking the child up during the episode C. allowing the child to roam about without supervision D. talking to the child while the episode is occurring

A. allowing the episode to take its course Rationale: Night terrors occur during the early hours of sleep. The child does not fully awaken and rarely remembers the night terror in the morning. For night terrors, instruct the family to let these episodes take their course while providing a safe environment and making sure the child does not harm oneself in any way. They should not interact with the child because the child will not have any understanding of who or what is around him or her. Because full consciousness has not been reached, the child may interpret a helpful action as a harmful one and may strike out or hurt himself or herself during the process.

The nurse is teaching a 14-year-old child on the proper use of a metered-dose inhaler to control symptoms of asthma. Which teaching points should the nurse include in these instructions? Select all that apply. a. Take two puffs at a time. b. Shake the canister before using. c. Wait 5 minutes between puffs. d. Hold the breath for 5 to 10 seconds. e. Activate the inhaler while taking a deep breath.

Answer: B, C Rationale: A DPI is a powder medication and should not be shaken prior to administration. Corticosteroids can cause an oral fungal infection. The client should rinse and expectorate following medication administration. The client should breathe in slowly (about 3 to 5 seconds) to administer the medication into the lungs. After inhalation of the medication, the client should hold his breath for 5 to 10 seconds. The client should wait 1 min between puffs.

The mother of a school-age child is distraught because the child has been diagnosed with obesity. What actions should the nurse suggest to the mother to help the child with this problem? Select all that apply. A. Explain that obesity will lead to an early death. B. Maintain a balanced eating approach in the home. C. Purchase books explaining the latest ways to lose weight. D. Seek out a preteen weight loss group for the child to participate. E. Encourage increased activity such as walking the dog after school.

B. Maintain a balanced eating approach in the home. D. Seek out a preteen weight loss group for the child to participate. E. Encourage increased activity such as walking the dog after school. Rationale: Strategies to help the school-age child with obesity include maintaining a healthy eating approach in the home, seeking a weight loss group with other preteens for the child to attend, and encouraging increased activity. Explaining that obesity will lead to an early death could cause the child to become obsessed with dieting and create an eating disorder. The child should not be encouraged to use fad diets to lose weight.

A 12-year-old child tells the school nurse, "I do not understand why my parents will not allow me to go to concerts without chaperones like some of my friends' parents. I feel like a baby compared to my friends." How will the nurse respond? A. "Your parents are right. Twelve years old is too young to be attending concerts without a chaperone." B. "I'm sure your parents are just very worried that you could get into trouble attending concerts at a young age." C. "Have you given any thought to why they don't let you go without a chaperone?Let's talk about some of the reasons they feel this way." D. "You are so young that you have plenty of time to go to concerts alone. Your parents just care about you."

C Rationale: Keeping dialogue open with the child and encouraging exploration of feelings isbeneficial and therapeutic communication. This will allow the child to identify reasonswithout being told why the parents have this rule. Peer pressure can be difficult for children tomanage. Helping them identify reasons for the parents' actions can help. Telling the child howto feel or that the parents only care doesn't allow conversation

The nurse takes a call from a concerned parent whose infant received routine immunizations the day before and now has a temperature of 101F (38.3C), is fussy, and pulling at the injection site. The parent wants to know what she should do. Which is the best response from the nurse to this parent? A. "You need to bring the infant to the emergency department to be sure the infant is not having an allergic reaction." B. "All infants have similar reactions but you should call back if the infant is still fussy in 24 hours." C. "This is a common reaction. Give your infant acetaminophen, cuddle your infant, and apply a cool compress to the injection site." D. "You can give your child ice-cold fluids and cover the injection site so that the infant does not scratch the site and get it infected."

C. "This is a common reaction. Give your child acetaminophen, cuddle him, and apply a cool compress to the injection site." Rationale: Adverse reactions vary with the type of immunization but usually are minor in nature. The most common adverse reaction is a low-grade fever within the first 24 to 48 hours and possibly a local reaction such as tenderness, redness, and swelling at the injection site. The infant may be fussy and eat less than usual. These reactions are treated symptomatically with acetaminophen for the fever and cool compresses applied to the injection site. The infant is encouraged to drink fluids but not necessarily ice-cold fluids. Holding and cuddling are comforting to the infant. These reactions may last longer than 24 hours and should subside. These are not signs of an allergic reaction. There is no need to cover the site.

The nurse is providing anticipatory guidance to a parent to help promote healthy sleep for the 3-week-old newborn. Which recommended guideline will the nurse include in the teaching plan? A. A stuffed animal may be placed in the crib for comfort once the newborn is 1 month old. B. Wrap the newborn in a blanket before placing in the crib for the night. C. Place the newborn on the back when sleeping during the day or night. D. Once sleeping through the night, continue to wake the infant up for night feedings.

C. Place the newborn on the back when sleeping during the day or night. Rationale: Sudden infant death syndrome (SIDS) has been associated with prone positioning of newborns and infants, so the newborn and infant should always be placed on the back to sleep. Nothing should be placed in the crib or bassinet except for the infant—no blankets, stuffed animals, pillows, etc. By 4 months of age night waking may occur, but the infant should be capable of sleeping through the night and does not require a night feeding.

The nurse is collecting information from a 14-year-old adolescent who was brought to the clinic for a well-child visit by the parent. Which techniques would be most beneficial in this process? Select all that apply. A. Ask the adolescent's parent to leave the room during the data collection and assessment. B. Direct the questions to the adolescent's parent but encourage the adolescent to interject as desired. C. Speak directly to the adolescent when making inquiries. D. Sit down facing the adolescent during the data collection period. E. Allow the adolescent's parent to be present during the collection of the health history but ask that the parent leave during the physical examination.

C. Speak directly to the adolescent when making inquiries. D. Sit down facing the adolescent during the data collection period. Rationale: When caring for teens it is important to establish a rapport. Questions should be directed to the teen. Sitting when obtaining the health history information is beneficial. It helps to promote comfort between the nurse and client. Parents and teens should be allowed to choose who is present during the physical examination.

A 12-year-old boy arrives at the emergency room experiencing nausea, vomiting, headache, and seizures. He is diagnosed with bacterial meningitis. Other findings include a decrease in urine production, hyponatremia, and water intoxication. Which pituitary gland disorder would be most associated with these symptoms? Diabetes insipidus Hyposecretion of somatotropin Syndrome of inappropriate antidiuretic hormone Hypersecretion of somatotropin

Syndrome of inappropriate antidiuretic hormone Rationale: Syndrome of inappropriate antidiuretic hormone (SIADH) is a rare condition in which there is overproduction of antidiuretic hormone by the posterior pituitary gland. This results in a decrease in urine production and water intoxication. As sodium levels fall in proportion to water, the child develops hyponatremia or a lowered sodium plasma level. It can be caused by central nervous system infections such as bacterial meningitis. As the hyponatremia grows more severe, coma or seizures occur from brain edema. Diabetes insipidus is characterized by polyuria, not decreased urine production. Hyposecretion of somatotropin, or growth hormone, results in undergrowth; hypersecretion results in overgrowth.

The nurse is assessing a 10-month-old infant at a well-child visit. Based on the report by the child's caregiver, which finding requires further follow up by the nurse? The infant was able to lift the head 45 degrees while prone 8 months ago but was unable to 1 month earlier. The infant was able to roll over 5 months ago and was able to sit independently 1 month later. The infant was able to pull to stand up 1 month ago but is still unable to stand up without pulling. The infant was able to walk independently 1 month ago but now requires the caregiver to hold the infant's hands.

The infant was able to walk independently 1 month ago but now requires the caregiver to hold the infant's hands. Rationale: The infant being able to walk independently 1 month ago but now requiring the caregiver to hold the infant's hands requires further follow up by the nurse, because this indicates that the infant passed a developmental milestone but is now losing that progress. Being able to roll over at 5 months old, being able to sit independently at 6 months old, being able to lift the head at 45 degrees while prone at 2 months old, and being able to pull to stand up at 9 months old, are all expected findings that do not require additional follow up by the nurse.

In providing anticipatory guidance related to choking hazards for infants, what should the nurse include in the teaching? Select all that apply. A. propping a bottle B. raw carrots C. shape sorter D. plastic bags E. stuffed animals

A. propping a bottle B. raw carrots D. plastic bags

A 5-year-old girl who was already admitted to the hospital for an unrelated condition suddenly becomes irritable, restless and anxious. These may be early signs of respiratory distress in a child if accompanied by: retractions. clubbing of fingers tachypnea. cyanosis.

tachypnea. Rationale: Restlessness, irritability, and anxiety result from difficulty in securing adequate oxygen. These might be very early signs of respiratory distress, especially if accompanied by tachypnea (an increased respiratory rate). Retractions can be a sign of airway obstruction but occur more commonly in newborns and infants than in older children. Cyanosis (a blue tinge to the skin) indicates hypoxia, which may be a sign of airway obstruction but would not be the first. Children with chronic respiratory illnesses often develop clubbing of the fingers, a change in the angle between the fingernail and nailbed because of increased capillary growth in the fingertips. Clubbing would not occur in an acute airway obstruction, as is indicated in the scenario above.

The nurse has completed an education session with parents of children diagnosed with food allergies. Which statement by a parent would indicate a need for additional education? "I will make sure my child always has the epinephrine auto-injector at all times." "If we need to use the epinephrine auto-injector we will need to notify the health care provider's office the next business day." "The grey part of the epinephrine auto-injector should never be removed until right before we use it." "I have found a website that makes medical alert bracelets in my child's favorite color."

"If we need to use the epinephrine auto-injector we will need to notify the health care provider's office the next business day." Rationale: If an epinephrine auto-injector is used, the child still needs immediate medical attention. An epinephrine auto-injector should be carried with the child at all times. When administering an epinephrine auto-injector, the grey safety cap should not be removed until immediately prior to use. Medical alert bracelets or necklaces should be worn by all children with severe allergies.

An adolescent client has just been diagnosed with systemic lupus erythematosus (SLE). Following client education about the disease, which statement by the client demonstrates understanding of SLE? "SLE is a rheumatic disease that mostly affects my joints." "SLE is an autoimmune disorder that I will always have, with times of flare- ups and times of minimal to no symptoms." "SLE only affects my skin. It seldom causes problems in any other organs." "If my SLE has been found early enough in the disease process, there is a good chance that medication can cure it."

"SLE is an autoimmune disorder that I will always have, with times of flare- ups and times of minimal to no symptoms." Rationale: SLE is a systemic autoimmune disease that can affect any organ system, including the skin. There is no cure for SLE, but with proper treatment and if the client cares for themselves properly, the disease can have periods of remission and fewer flare-ups.

The nurse is educating the family of a child receiving an infusion of intravenous immunoglobulin (IVIG) and high-dose aspirin therapy during the acute phase of Kawasaki disease. What statement(s) by the parent indicates additional teaching is needed? Select all that apply. "I will monitor my child for excessive bleeding and notify the health care provider if I notice any." "We have an appointment next week for the varicella vaccine to be given." "I worry day care will expose my child to measles, so I will have my child vaccinated soon." "I plan to help my child's lips by applying a petroleum jelly or similar lubricant." "I will be sure to maintain follow-up appointments at the cardiologist's office."

"We have an appointment next week for the varicella vaccine to be given." "I worry day care will expose my child to measles, so I will have my child vaccinated soon." Rationale: In the acute phase, high-dose aspirin and intravenous immunoglobulin (IVIG) are given to manage Kawasaki disease. The parent would be taught to avoid vaccines, especially varicella and measles, mumps, rubella vaccine, for several months following IVIG infusion due to the IVIG preventing the body from building antibodies and rendering the vaccines ineffective. The parent would be correct in monitoring for bleeding secondary to the high-dose aspirin, keeping the child's lips lubricated, and keeping cardiology appointments to monitor for cardiac complications from Kawasaki disease.

An adolescent who is depressed states, "Nothing ever seems to be right in my life." Which would be the most appropriate response by the nurse? "Things will be better when you go off to college." "You are feeling sad right now. It's a hard time." "Try to look at the bright side of things." "Being a teenager is hard work."

"You are feeling sad right now. It's a hard time." Rationale: Some degree of depression is present in most adolescents because they are not only losing their parents while they grow apart from them but also their carefree childhood. When using therapeutic communication, it is important for the nurse to accept the client's verbalization as real. Support should be real. Telling the adolescent that things will be better in college provides false reassurance. Telling the adolescent to "look on the bright side of things" or that "being a teen is hard work" offer platitudes and interrupt the client's interactions.

The nurse is speaking with the parent of a premature infant who is 6 months old. The parent asks the nurse when to begin feeding the infant solid food. The infant was born at 28 weeks. What is the best response by the nurse? A. "You can start feeding solid food to your infant when the infant turns 9 months old." B. "We generally start solid foods at 6 months so you can start feeding your infant solid food now." C. "Since your infant was born prematurely, you should wait until the infant is 12 months old to start solid food." D. "You can start feeding your infant solid food when the infant can hold the bottle on one's own."

A."You can start feeding solid food to your infant when the infant turns 9 months old." Rationale: All anticipatory guidance related to nutrition is based on the infant's corrected age. In other words, begin solids at 6 months corrected age, not chronological age, and delay the addition of whole milk until 12 months corrected age, rather than 1 year chronological age. This infant's corrected age is 3 months. The child will be 9 months old when the corrected age is 6 months.

The parent of a 2-year-old child asks the nurse, "How can I help to foster my child's language skills?" Which suggestions by the nurse would be appropriate? Select all that apply. A. accepting the child's "no" as indicating actual refusal B. reading to the child frequently C. naming objects as they are used with the child D. answering the child's questions in simple terms E. supplying things to the child before he or she asks for them

B. reading to the child frequently C. naming objects as they are used with the child D. answering the child's questions in simple terms Rationale: To foster language development, parents need to know that the child's "no" does not necessarily mean that the child is refusing. The child may use the word because he or she does not understand it or is practicing a sound that has a strong effect on those around the child. Reading often, naming objects as they are used, and answering the child's questions in simple terms help to foster the understanding that words are not just meaningless sounds. Providing opportunities for the child to ask for things also helps to foster language development by encouraging the child to use language.

The nurse receives the shift report of multiple pediatric clients. Which pediatric client will the nurse see first? A. a child with history of hypertension and a current blood pressure of 130/90 mm Hg B. an infant whose parents report difficulty feeding with a temperature of 100.1°F (38°C) C. a toddler with tetralogy of Fallot squatting quietly in the corner of the room D. an adolescent with coarctation of the aorta with reports of coughing and coryza

C. a toddler with tetralogy of Fallot squatting quietly in the corner of the room Rationale: The first child the nurse will see is the child showing signs and symptoms of decreased pulmonary blood flow and possible hypercyanotic (tet) spell, which includes a toddler with tetralogy of Fallot squatting. Squatting increases systemic vascular resistance and forces blood to flow through the narrow pulmonary valve to improve oxygenation. An infant with difficult feeding and an elevated temperature may have an infection but could be seen after addressing a potential respiratory/circulatory issue. The child with history of hypertension who has an elevated blood pressure can be seen later because this is an expected finding and not life-threatening. The adolescent with coarctation of the aorta being seen for coughing and coryza without any other signs of distress can also be seen later.

A 15-year-old male complains of persistent scrotal pain, edema, and nausea since being hit in the groin by a baseball 3 hours ago. Which is the priority action by the nurse? Ensuring that the teen is assessed by the physician immediately Documenting the swelling and discoloration Applying an ice pack to alleviate the pain Administering pain medications as ordered

Ensuring that the teen is assessed by the physician immediately Rationale: The nurse should suspect testicular torsion, which requires emergent interventions to prevent testicular death. Ice should never be applied because of the decreased circulation. Documentation of signs and administration of pain medications are both correct but are not the highest priority tasks.

A 12-year-old child has broken the arm and is showing signs and symptoms of shock. Which action will the nurse take first? Begin hyperventilation. Draw blood for type and crossmatch. Provide oral analgesics as ordered. Establish a suitable IV site.

Establish a suitable IV site. Rationale: The goal of treating shock is to restore circulating blood volume. This requires that vascular access be obtained to administer fluids and vasoactive drugs. Hyperventilation is reserved for temporary treatment of severe intracranial pressure. Analgesics should not be administered prior to neurologic and cardiovascular examination being performed. Drawing blood for type and crossmatch would be once vascular access is obtained and fluid and drug therapy has been initiated.

1230: A 7-year-old child is admitted to the ED with severe vomiting andd diarrhea for 2 days. The parents report that the child was unable to keep down water for the past 8 hours. Vitals: T: 99.8F HR: 120 beats/min R: 32 B/P: 160/80mmHg The child reports intermittent abdominal pain (7/10) An IV was started in R. arm with a total of 3 boluses started 1530: The client is sleepy but rouses when called by name. Vitals: T: 100.7F HR: 138 beats/min R: 34 B/P: 98/55mmHg Urine output over the last 2 hours: 22mL A nurse has charted the above note. How will the nurse interpret the assessment findings? The child is experiencing the onset of refractory shock. The child's change in vital signs indicates septic shock. The child is exhibiting signs of decompensated shock. The child's urine output is an indicator of obstructive shock.

The child is exhibiting signs of decompensated shock. Rationale: This child is suffering from hypovolemia. The increase in the child's heart rate, decrease in blood pressure, lethargy, and urine output are signs of decompensation of hypovolemic shock. Signs of refractory (irreversible) shock include extreme hypotension, bradycardia, and evidence of end-organ damage. Septic shock is a form of distributive shock resulting from overwhelming infection. Obstructive shock results from a mechanical blockage of blood into the heart and major vessels.

The nurse encourages the mother of a toddler with acute laryngotracheobronchitis to stay at the bedside as much as possible. The nurse's rationale for this action is primarily that: Mothers of hospitalized toddlers often experience guilt. The mother can provide constant observations of the child's respiratory efforts. Separation from the mother is a major developmental threat at this age. The mother's presence will reduce anxiety and ease the child's respiratory efforts.

The mother's presence will reduce anxiety and ease the child's respiratory efforts. The family's presence will decrease the child's distress. The mother may experience guilt, but this is not the best answer. Although separation from the mother is a developmental threat for toddlers, the main reason to keep parents at the child's bedside is to ease anxiety and therefore respiratory effort. The child should have constant cardiorespiratory monitoring and noninvasive oxygen saturation monitoring, but the parent should not play this role in the hospital.

A 12-year-old girl has recently begun menstruating and is well into puberty. The child is visiting the health care provider today for a routine physical examination. Which finding should cause concern in the nurse? vulvar irritation irregular periods breasts of slightly different sizes supernumerary nipple

Vulvar irritation Rationale: Any secretions that cause vulvar irritation should be evaluated by a health care provider because this does suggest infection. Most girls have some menstrual irregularity during the first year or two after menarche (the start of menstruation). Breast development is not always symmetrical, so it is not unusual for a girl to have breasts of slightly different sizes. After the condition has been checked during a physical examination to assure that no tumors are present to make one breast larger or that the other is diseased in some way to make it smaller, the child can be reassured this development is normal. Supernumerary (additional) nipples may darken or increase in size at puberty. Be sure girls understand a supernumerary nipple is affected by the hormones in the body in the same way as other breast tissue, so the child isn't frightened by the accessory nipple enlarging with puberty or in a future pregnancy.

The adoptive parents of a child who is 7 years old and HIV positive are concerned about telling their child about the condition. What information can be provided by the nurse? When providing health information to a child of this age it should be simplistic and at the child's level of understanding. The child should not have information about the child's health provided at this age. Once a child is apprised of the health concerns the child does not normally experience any aftereffects. A child at this age should have full disclosure of the condition.

When providing health information to a child of this age it should be simplistic and at the child's level of understanding. Rationale: When children have a chronic condition they often realize that they have special concerns even before they are fully able to understand them. Information should be provided that is developmentally appropriate. Excessive information and details should be limited. Children who have this type of information may experience anger, depression, and difficulty in school.

The nurse has received morning report on a group of pediatric clients. Which pediatric client will the nurse see first? an infant with rhinorrhea, coughing, and oxygen saturation of 92% a toddler with a temperature of 100.1°F (38°C), and a harsh, barking cough a school-age child with dysphagia, drooling, and a hoarse voice a preschool child with crackles in the right lower lobe and chest pain

a school-age child with dysphagia, drooling, and a hoarse voice Rationale: The child with signs and symptoms of epiglottitis should be seen first because epiglottitis is an emergency that can quickly cause airway obstruction. A child with signs of bronchiolitis with an oxygen saturation of 92% is more stable than this child with epiglottitis. A toddler with signs of croup is more stable than this child with epiglottitis. A child with signs and symptoms of pneumonia is more stable than this child with epiglottitis.

A nurse working at a pediatric clinic includes an adolescent history for every child aged 11 to 18 years when the child is new to the office. Which issues should the nurse address when the parents are not in the room? Select all that apply. job responsibilities abuse/domestic violence self-image sexual activities adolescent's fit in the family

abuse/domestic violence sexual activities Rationale: A primary reason for addressing abuse and sexual activities while the parent is not in the room is that the child may not want the parent/family member to know about sexual activity; the provider may be able to help the child develop ways of telling the parent about the issue or the child may divulge the "real" reason for coming to the health care agency. The child's self-image, job responsibilities, and his or her fit in the family could all be assessed while the parent/family member is in the room.

Diabetes insipidus is a disorder of the posterior pituitary resulting in deficient secretion of which hormone? luteinizing hormone adrenocorticotropic hormone thyroid stimulating hormone antidiuretic hormone

antidiuretic hormone Rationale: Central diabetes insipidus (DI), also called neurogenic, vasopressin-sensitive, or hypothalamic DI, is a disorder of the posterior pituitary that results from deficient secretion of ADH. Nephrogenic DI is a result of the inability of the kidney to respond to ADH.

An adolescent is found wandering around. The client is confused, sweaty, and pale. Which test will the nurse prepare to perform first? computed tomography (CT) scan serum ketone testing blood toxicology blood glucose level

blood glucose level Rationale: It is important to determine a blood glucose level on the adolescent because the adolescent is exhibiting signs of hypoglycemia and needs to be treated quickly. Serum ketone testing would be indicated if the adolescent were exhibiting symptoms of hyperglycemia. A CT scan or toxicology test may be needed if the adolescent's glucose level were within normal range.

A child diagnosed with a ventricular septal defect (VSD) is seen in the clinic. The nurse conducts an assessment and reviews the child's recent laboratory results. The nurse suspects the child may have developed heart failure. Which finding is the best indicator of the nurse's suspicion? low hematocrit crackles on lung auscultation poor weight gain reports of increased fatigue

crackles on lung auscultation Rationale: The finding that is the best indicator that the child has developed heart failure is crackles on auscultation of the lungs. This indicates pulmonary congestion and is a progressive finding in heart failure. Reports of increased fatigue and lack of weight gain are related to the VSD but do not necessarily indicate the development of heart failure. Anemia could also be associated with the VSD because it relates to poor feeding and lack of weight gain.

The adolescent comes to the clinic seeking information about sexuality concerns. The clinic nurse assures the adolescent that confidentiality and privacy will be maintained unless a life-threatening situation occurs. Maintaining confidentiality demonstrates which nursing goal? Select all that apply. development of a trusting relationship compliance with existing laws inappropriate response because adolescents are minors an environment where adolescents can be truthful concern from parents who pay the office visit bill

development of a trusting relationship compliance with existing laws an environment where adolescents can be truthful Rationale: Adolescents may seek a health care appointment for an unrelated health concern as a reason to discuss a sexual health question with a health care professional. Reassurance should be given to the adolescent that all questions and concerns will be addressed and will be kept confidential. This is the basis for the nurse-client relationship. All questions and concerns do not involve treatment and therefore do not involve parental consent. Parents may voice concerns because they are responsible for the insurance and billing. The nurse should act as a client advocate and work with the parents to develop a mutual understanding of the situation.


Related study sets

Network Auth and Security Chapter 14

View Set

NU 112- Nursing Concepts (REDUCTION OF RISK POTENTIAL)

View Set

Chapter 5 - Canvas Quiz Answers (plus written questions)

View Set

Le Petit Prince: Chapitre 4 Questions

View Set

frl chapter 9, chapter 10, chapter 12

View Set

Driver's Ed Sections 3.7 and 3.8

View Set